Search found 46 matches


(1) The example you presented is correct, up until: "= squareroot(4)= +- 2" On the GMAT, anything under a square root sign must yield a positive value. Negative Square roots are imaginary numbers and are outside the scope of the GMAT. Therefore squareroot(4) simply yields a positive value ...

by rros0770

Wed Apr 27, 2011 8:27 am
Forum: Problem Solving
Topic: Gmat prep problems
Replies: 3
Views: 1199

Thanks Anshu, you made that look too easy. I see where I was getting hung up now

by rros0770

Thu Mar 10, 2011 7:31 pm
Forum: Problem Solving
Topic: Angela's family drank
Replies: 8
Views: 2907

I'm coming up with [spoiler]5 (inluding Angela)[/spoiler] I solved it by selecting/plugging in values for the coffee and milk: 6 ounces coffee 4 ounces milk (10 ounces of milk+coffee total) Using the above selected values: 1/6 coffee + 1/4 milk = B-ounces 1 ounce coffee + 1 ounce milk = 2 ounce mixt...

by rros0770

Thu Mar 10, 2011 6:55 am
Forum: Problem Solving
Topic: Angela's family drank
Replies: 8
Views: 2907

Hi Amit-

Anurag did exclude 121 and 81

By subtracting 1 from the difference between 121 - 81, we are excluding the two extremes from the total count

by rros0770

Tue Mar 08, 2011 3:58 pm
Forum: Problem Solving
Topic: √n differ from √100
Replies: 3
Views: 1126

Hey Neilcao- Agree with the points Night Reader made. Is this copied word for word also? We're given percentages in the first half of the stem and in both statements, but the 2nd half of the stem asks " ...how many people choose...". It wants a number, which we can't provide from this info...

by rros0770

Wed Feb 23, 2011 1:54 pm
Forum: Data Sufficiency
Topic: 80% choose course M and C.....
Replies: 2
Views: 944

Got it- I see where our opinions differ here Night Reader: Quoting you: ">=4 means {4,5,6} and =<2 means {1,2} it can also be the case that none of the projects has 3 employees assigned. Please show me precise condition for 3. " I believe the 2nd statetment would encompass none (0) ,1 or 2...

by rros0770

Fri Feb 11, 2011 11:33 am
Forum: Data Sufficiency
Topic: ds percents and time distance
Replies: 14
Views: 2910

25% assigned to either 4, 5, 6, 7, 8, 9, 10, 11....etc 35% assigned to 0, 1 or 2 40% left unnaccounted for. Median would be the 50% mark, which would then fall under the 3 employee category I'm under the impression it has to be 3 employees, I can't derive any other scenario in which this would not b...

by rros0770

Fri Feb 11, 2011 11:25 am
Forum: Data Sufficiency
Topic: ds percents and time distance
Replies: 14
Views: 2910

Agree it's C

25% have 4+
35% have 1 or 2

40% of the projects left unaccounted for.

Would have to be 3 employees. If no project had 3 employees assigned to it then 40% of the projects would be left uaccounted for

by rros0770

Fri Feb 11, 2011 10:55 am
Forum: Data Sufficiency
Topic: ds percents and time distance
Replies: 14
Views: 2910

Hey Night Reader- I follow your logic, just missing why we've ruled out (A) I agree with the calc you set up here: "2n/3=i {i is integer} +1/3" From the stem: (1)We know that (n) is not negative. (2)We know that (n) is not zero, because if (n)= 0 then we get the below, and your calc wouldn...

by rros0770

Fri Feb 11, 2011 10:32 am
Forum: Data Sufficiency
Topic: Divisiblity and remainder help
Replies: 12
Views: 3886

alltimeacheiver- What's the OA ? Is there a Choice (E) None? As there were only 4 choices posted below I think A is the answer. Let me know if I missed something though: We're told in the question stem that n is nonnegative (zero or positive) and 2n/3 leaves a remainder of 1, so: n= 3(k) + 1 In orde...

by rros0770

Fri Feb 11, 2011 9:48 am
Forum: Data Sufficiency
Topic: Divisiblity and remainder help
Replies: 12
Views: 3886

Agree with ManSab.

JK-

"I have Y=45, EDC = 45, ECD = 90"

This would be the case if CD=CE


Question states that CE=DE
Therefore EDC and ECD are the angles that are equal to each other, so:
Y= 45, EDC = 67.5, ECD = 67.5

Hope that helped.

by rros0770

Fri Jan 21, 2011 10:16 am
Forum: Problem Solving
Topic: Triangles
Replies: 5
Views: 5118

Tried solving for both combos and for perms because was confused about the answer choices:

I'm getting 14 if order doesn't matter
If order matters I'm getting the same as Junegmat with 40

Don't think I missed anything....
OA?

by rros0770

Sat Dec 04, 2010 1:54 pm
Forum: Problem Solving
Topic: combinations /600-650
Replies: 5
Views: 1506

Yep, dead on with needing to factoring it.

Check this post:

http://www.beatthegmat.com/gmat-prep-ex ... 17656.html

by rros0770

Tue Nov 30, 2010 3:30 pm
Forum: Problem Solving
Topic: Simplify
Replies: 1
Views: 953

If X-Y > 10, is x-y > x+y? Statement 1: x=8 Plug 8 into the first equation: 8-Y > 10 In order for 8-Y to be greater then 10, Y must be a negative number. Plug 8 and (-Y) into the second equation: 8 - (-Y) > 8 + (-Y), this becomes: 8+Y > 8-Y [spoiler]From this we can see that 8+Y is greater the 8-Y S...

by rros0770

Tue Nov 30, 2010 2:34 pm
Forum: Problem Solving
Topic: DS Question
Replies: 2
Views: 1029

Hey GDK- It's providing us the average cost of the 6 items, not the actual cost of the 6 items. 5 items could cost $1 each and the 6th item could cost $235, and that will average out to $40. In the above scenario, there would be tax on the $125 coat and tax on that $235 item which would total $25.20...

by rros0770

Fri Nov 26, 2010 12:20 pm
Forum: Data Sufficiency
Topic: OG 12 DS question
Replies: 4
Views: 1581